Proof that norm of submatrix must be less than norm of matrix it's embedded in

Click For Summary
The discussion focuses on proving that the norm of a submatrix B is less than the norm of the larger matrix A it is derived from. The relationship ||B|| = ||M_1 * A * M_2|| leads to the inequality ||B|| <= ||M_1|| * ||A|| * ||M_2||, indicating that the norms of the matrices M_1 and M_2, which are derived from the dimensions of A, must be considered. It is suggested that if B is a 1x1 matrix, M_1 and M_2 would be matrices with limited non-zero entries, ultimately leading to a maximum norm of 1. The discussion emphasizes the importance of identifying M_1 and M_2 explicitly to apply the norm inequality effectively. A clear understanding of block matrices and their properties is crucial for completing the proof.
Simfish
Gold Member
Messages
811
Reaction score
2

Homework Statement



http://dl.dropbox.com/u/4027565/2010-10-10_194728.png

Homework Equations





The Attempt at a Solution




||B|| = ||M_1 * A * M_2 ||

So from an equality following from the norm, we can get...

||B|| <= ||M_1||*||A||*||M_2||.

Now, we know that B is a submatrix of A. So if A is 4x3, then M_1 must be 1x4 and M_2 must be 3X1 (I know that block matrices are more complicated than that, but this might work). What this also means is that the combined product of M_1 and M_2 must be <= 1. But beyond that, I'm stuck. Is there another step I should take?

Thanks!
 
Last edited by a moderator:
Physics news on Phys.org
Okay so if B is ultimately 1x1, then M_1 and M_2 must both be matrices with 0s everywhere except for one row (or column). So the maximum norm (under any situation) would be 1.
 
You cane always embed B in 4x4 block matrices by adding 0 and I as appropriate blocks. Block matrices M_1 and M_2 will probably be orthogonal projections of norm ||M||=1. In any case the key is to find explicitly M_1 and M_2 and their norms. Then use ||ABC||\leq ||A||\,||B||\,||C||.
 
Last edited:
Question: A clock's minute hand has length 4 and its hour hand has length 3. What is the distance between the tips at the moment when it is increasing most rapidly?(Putnam Exam Question) Answer: Making assumption that both the hands moves at constant angular velocities, the answer is ## \sqrt{7} .## But don't you think this assumption is somewhat doubtful and wrong?

Similar threads

Replies
2
Views
6K
Replies
7
Views
6K
Replies
7
Views
9K
Replies
1
Views
3K
  • · Replies 4 ·
Replies
4
Views
4K
  • · Replies 11 ·
Replies
11
Views
4K
  • · Replies 7 ·
Replies
7
Views
2K
  • · Replies 14 ·
Replies
14
Views
10K
  • · Replies 2 ·
Replies
2
Views
2K
  • · Replies 6 ·
Replies
6
Views
4K